A hobby store prices model train track using a proportional relationship between the length of track (in inches) and the cost in dollars.

If 6.4
6
.
4
inches of track costs $16
$
16
, what is the constant of proportionality?

A Hobby Store Prices Model Train Track Using A Proportional Relationship Between The Length Of Track

Answers

Answer 1

Answer:

If... 6.4 inches : 16 dollars

Then... 32 inches = 80 dollars.  

And, 1 inch of track = 80/32 dollars.

80/32 = 2.5.

So, the answer is: 1 inch of track costs 2.5 dollars.

Answer 2

The  constant of proportionality is $2,50.

The equation used to represent direct proportionality is: y = kx

Where:

y = dependent variable

x = independent variable

k = constant of proportionality

Here, the dependent variable is the cost of the track. The independent variable is the length of the tracks.

$16 = 6.4k

k = 16 / 6.4 = $2.5

A similar question was answered here: https://brainly.com/question/17033082


Related Questions

Given: AB tangent at D, AD = OD = 4 Find: Area of the shaded region

Answers

Answer:

1.72

Step-by-step explanation:

AB tangent at D, AD = OD = 4

so triangle OAD is right angle with side of 4 and 4.

area of OAD = 1/2 * 4 * 4 = 8

Angle AOD = DAO = 45 deg.

so circular sector OCD area = area of circle O * 45/360

= pi * 4 * 4 * 45/360

= 2pi

Shade area ACD = trigangle OAD - circular sector OCD

= 8 - 2pi

= 1.72

URGENT ! HELP ME I WILL MARK YOU BRAINLIEST !!!!

pleasee fasterrr !!!!​

Answers

Answer:

3b (3a - 4b)

Step-by-step explanation:

Answer:

C

Step-by-step explanation:

9ab - 12b² ← factor out 3b from each term

= 3b(3a - 4b) → C

use order operation

Answers

Answer:

The correct answer is the final option, 2.

Step-by-step explanation:

[(16+4) - 10] - 8

Our first step in this problem, according to the order of operations, is to simplify what is inside the parentheses.  Since there are multiple sets of parentheses, we must simplify the innermost group first.  If we add 16 and 4, we get:

[20 - 10] - 8

Next, we must perform the subtraction inside the parentheses.  This gives us:

10 - 8

Now, we can perform the final operation.  

10 - 8 = 2

Therefore, the correct answer is 2.

Hope this helps!

Given quadrilateral ABCD, where the diagonals AC and BD intersect at point E. AE≅EC and BE≅DE Can you prove can you prove that the figure is a parallelogram? Explain. A. Yes; two opposite sides are both parallel and congruent. B. Yes; diagonals of a parallelogram bisect each other. C. Yes; opposite sides are congruent. D. No; you cannot determine that the quadrilateral is a parallelogram.

Answers

Answer:

B. Yes; diagonals of a parallelogram bisect each other.

Step-by-step explanation:

The diagonals of a parallelogram bisect each other. If the diagonals of a quadrilateral bisect each other, the quadrilateral is a parallelogram.

Answer: B. Yes; diagonals of a parallelogram bisect each other.

If AE=EC and BE=DE then quadrilateral ABCD is a parallelogram because  diagonals of parallelogram bisect each other.

What is parallelogram?

A parallelogram is a 2 dimensional figure whose opposite sides are equal to each other and parallel to each other. Area of parallelogram is base*height.

How to prove quadrilateral a parallelogram?

To be parallelogram ΔAEB and ΔEDC should be congruent.

Angle AEB= Angle DEC

AE=EC

DE=EB

so both triangles are congruent by side, side, angle.

Similarly AE=EC , DE=EB and vertical angles AED= Vertical angle BEC.

Therefore triangle AED and BEC are congruent and that makes all their corresponding sides are also congruent.

And AB=DC.

Hence both pairs of opposite sides of a quadrilateral are congruent ,then the quadrilateral is a parallelogram.

Learn more about parallelogram at https://brainly.com/question/970600

#SPJ2

Find the LCM of
[tex]14 {a}^{3} {b}^{2} [/tex]
and
[tex]20 {a}^{4} {b}^{2} [/tex]

Answers

Answer:

Hi,

I hope you are searching for this answer.

The lcm of an expression is calculated by taking the common factor and multiply the common factor with the remaining one, alright.

or, simply the lcm of an expression is calculated by the product of common factors and the factors which are not common.

Hope it helps...

Julie had 2730 cards and Kim had 3570 cards at first.

Julie gave some of her cards to Kim. In the end, Kim had thrice as many cards as Julie.

How many cards did Julie give Kim.​

Answers

Answer:

1155

Step-by-step explanation:

Total number of cards is 2730+3570=6300

Since Kim now has 3 times the card of Julie so Julie must have 6300/4=1575.

So, Julie gave 2730-1575=1155

Answer:

1155 cards

Step-by-step explanation:

3(2730-x)=3570+x

8190 - 3x = 3570 + x

4620 - 3x = x

4620 = 4x

1155 = x

For which system of inequalities is (3,-7) a solution? A. x + y < -4 3x + 2y < -5 B. x + y ≤ -4 3x + 2y < -5 C. x + y < -4 3x + 2y ≤ -5 D. x + y ≤ -4 3x + 2y ≤ -5

Answers

Answer:

The correct option is;

D x + y ≤ -4, 3·x + 2·y  ≤-5

Step-by-step explanation:

A. For the system of inequality, x + y < -4, 3·x + 2·y  <-5

We have;

y < -4 - x, When x = 3, y < -7

y < -2.5 - 1.5·x, When x = 3, y = -7

B. For the system of inequality, x + y ≤ -4, 3·x + 2·y  <-5

We have;

y ≤ -4 - x, When x = 3, y ≤ -7

y < -2.5 - 1.5·x, When x = 3, y < -7

C. For the system of inequality, x + y < -4, 3·x + 2·y  ≤-5

We have;

y < -4 - x, When x = 3, y < -7

y ≤ -2.5 - 1.5·x, When x = 3, y ≤ -7

D. For the system of inequality, x + y ≤ -4, 3·x + 2·y  ≤-5

We have;

y ≤ -4 - x, When x = 3, y ≤ -7

y ≤ -2.5 - 1.5·x, When x = 3, y ≤ -7

Therefore, the system of inequality for which (3, -7) is a solution is D, x + y ≤ -4, 3·x + 2·y  ≤-5.

The graph of a linear system is shown



Answers

Answer:

The coordinates of the point of intersection is (2, 7)

Step-by-step explanation:

From the graph, we have the two intersecting straight lines each described as follows;

The first straight line graph from the left, we have;

y-intercept = (0, 5), x-intercept = (-5, 0) therefore, the slope, m = (0 - 5)/(-5 - 0) = 1

The equation of the graph is slope-intercept form is y - 5 = 1×(x - 0)

Which gives;

y = x + 5.........................(1)

The second lie graph has y-intercept  = (0, 9), x-intercept = (9, 0) therefore, the slope, m = (0 - 9)/(9 - 0) = -1

The equation of the graph in slope-intercept form is y - 9 = -1×(x - 0)

Which gives;

y = -x + 9.......................(2)

To find the point of intersection, we equate equation (1) to equation (2), which are the y-value function of x, we have;

x + 5 = -x + 9

x + x  = 9 - 5

2·x = 4

x = 4/2 = 2

x = 2

From either equation (1) or equation (2), we can find the value of y as follows;

y = -x + 9, where x = 2 gives;

y = -2 + 9 = 9 - 2 = 7

y = 7

The coordinates of the point of intersection is (2, 7).

How many solutions does the following equation have? 4(y-30)=4y+124(y−30)=4y+12

Answers

Answer:

A. No solution

Step-by-step explanation:

Choose 1 answer:

A. No solutions

B. Exactly one solution

C. Infinitely many solutions

Solution

Given:

4(y-30)=4y+12

Open parenthesis

4y-120=4y+12

Collect like terms

4y-4y=12+120

0=142

There is no solution to the equation, therefore, the answer is A

State if the triangles are similar. If so, how do you know they are similar and complete the similarity statement. Triangle LKJ≈____

Answers

Answer: C) similar, SAS similarity, triangle LQR

==============================================

Explanation:

The vertical angles KLJ and QLR are congruent. This forms the "A" in "SAS". The angles in question are between the marked sides.

KL = 18 is twice that of QL = 9, or put another way, KL/QL = 18/9 = 2. The ratio of the sides is 2. Also, JL/RL = 16/8 = 2 is the same ratio. Because both pairs of sides have the same ratio, the sides are in proportion. This helps form the two "S" letters of "SAS".

The original triangle has LKJ mentioned at the top. Note the order as its important. We start with L and move to K, so LK is the first segment mentioned. LK = 18 pairs up with LQ = 9, meaning that LQ must be the first segment mentioned of the answer triangle. Therefore LQR is the correct letter sequence if we start with point L. Writing QLR is not correct because Q is the first letter here but Q does not pair up with L.

La fuerza necesaria para evitar que un auto derrape en una curva varía inversamente al radio de la curva y conjuntamente con el peso del auto y el cuadrado de la velocidad del mismo. Supongamos que 400 libras de fuerza evitan que un auto que pesa 1600 libras derrape en una curva cuyo radio mide 800 si viaja a 50mph. ¿Cuánta fuerza evitaría que el mismo auto derrapara en una curva cuyo radio mide 600 si viaja a 60mph ?

Answers

Answer:

768 libras de fuerza

Step-by-step explanation:

Tenemos que encontrar la ecuación que los relacione.

F = Fuerza necesaria para evitar que el automóvil patine

r = radio de la curva

w = peso del coche

s = velocidad de los coches

En la pregunta se nos dice:

La fuerza requerida para evitar que un automóvil patine alrededor de una curva varía inversamente con el radio de la curva.

F ∝ 1 / r

Y luego con el peso del auto

F ∝ w

Y el cuadrado de la velocidad del coche

F ∝ s²

Combinando las tres variaciones juntas,

F ∝ 1 / r ∝ w ∝ s²

k = constante de proporcionalidad, por tanto:

F = k × w × s² / r

F = kws² / r

Paso 1

Encuentra k

En la pregunta, se nos dice:

Suponga que 400 libras de fuerza evitan que un automóvil de 1600 libras patine alrededor de una curva con un radio de 800 si viaja a 50 mph.

F = 400 libras

w = 1600 libras

r = 800

s = 50 mph

Tenga en cuenta que desde el

F = kws² / r

400 = k × 1600 × 50² / 800

400 = k × 5000

k = 400/5000

k = 2/25

Paso 2

¿Cuánta fuerza evitaría que el mismo automóvil patinara en una curva con un radio de 600 si viaja a 60 mph?

F = ?? libras

w = ya que es el mismo carro = 1600 libras

r = 600

s = 60 mph

F = kws² / r

k = 2/25

F = 2/25 × 1600 × 60² / 600

F = 768 libras

Por lo tanto, la cantidad de fuerza que evitaría que el mismo automóvil patine en una curva con un radio de 600 si viaja a 60 mph es de 768 libras.

In the figure shown, what fractional part of the circle is shaded?

1. 4/15
2. 3/8
3. 5/8
4. 11/15

Answers

Answer:

4/15

Step-by-step explanation:

Add up the unshaded part

1/3 +2/5

Get a common denominator

1/3 *5/5 + 2/5 *3/3

5/15 + 6/15

11/15

Subtract this from 1 for the entire circle

15/15 -11/15

4/15

4/15 is shaded

Answer:

answer is

[tex] \frac{4}{15} [/tex]

Step-by-step explanation:

explanation is in the picture!

Write (25x^2 + 30x + 12) in the form (ax + b)^2 + c.

Answers

Answer:

25 x^2 + 30 x + 12

Parcelas

Plots

Parcelas

Figura geométrica

parabola

Formas alternativas

x (25 x + 30) + 12

(5 x + 3)^2 + 3

5 x (5 x + 6) + 12

Raíces complejas

x = -1/5 i (sqrt(3) + -3 i)

x = 1/5 i (sqrt(3) + 3 i)

Step-by-step explanation:

Divide. Write your answer as a decimal.

8.722÷(−3.56)=​

Answers

Answer:

-2.45

Step-by-step explanation:

Answer:

-2.45

Step-by-step explanation:

Figure a is a scale image of figure b. Figure a maps to figure b with a scale factor of 0.75 What is the value of x? please answer asap!

Answers

Answer:

x = 7.5

Step-by-step explanation:

Step 1: Create a fraction with the known sides

[tex]\frac{x}{10}[/tex]

Step 2: Set the fraction equal to the scale factor

[tex]\frac{x}{10}=\frac{0.75}{1}[/tex]

Cross multiple to solve for x

[tex]x = 7.5[/tex]

Therefore x is equal to 7.5

Answer:

7.5

Step-by-step explanation:

did it on khan

An octagonal pyramid ... how many faces are there, how many vertices and how many edges? A triangular prism ... how many faces are there, how many vertices and how many edges? a triangular pyramid ... how many faces are there, how many vertices and how many edges?

Answers

1: 8 faces and 9 with the base 9 vertices and 16 edges

2: 3 faces and 5 with the bases 6 vertices and 9 edges

3: 3 faces and 4 with the base 4 vertices and 6 edges

Answer:.

Step-by-step explanation:.

Can u guys answer these 2 questions pls

Answers

Answer:

14)  answer is 4

15) proved

Step-by-step explanation:

x=7-4√3 , √x +1/√x

√(7-4√3) +1/√7-4√3)=

8-4√x/√(7-4√3)= 4 ( use calculator)

(81/16)^-3/4*[(25/9)^-3/2 ÷(5/2)^-3]=(81/16)^-3/4= (9²/4²)^-3/4=(4^6/4)/(9^6/4)=8/(27)=8/27(25/9)^-3/2= (9^(3/2))/(25^3/2)=27/125(5/2)^-3=2³/5³ =8/125put the number to find the result:8/27[(27/125)÷(8/125)=8/27[(27/125)×125/8)]=    125 in nominator and dinaminotor=18/27[27/8]=1 proved

Last week, 17 employees exceeded their sales quota, 13 employees met their sales quota, and 3 employees didn't meet their sales quota. Express the number of employees who exceeded their sales quota to the number of employees who didn't exceed their sales quota. Question 11 options: A) 3:13 B) 16:17 C) 17:16 D) 17:33

Answers

Answer:

17:16

Step-by-step explanation:

Number of employees exceeded their sales quota = 17

Number of employees met their sales quota = 13

Number of employees didn't exceed their sales quota = 3

Now, we need to find the ratio of the number employees who exceeded their sales quota to the number of employees who didn't exceed their sales quota.

 

. If the market price of a dozen eggs at the local farmers market is 72 cents per dozen, will Brenda make an economic profit? Explain how you determined your answer. (4 points)

Answers

Cost of dozen eggs = 72 cents

Cost of 1 egg = 72÷12

= 6cents

Please click thanks and mark brainliest if you like

Do these pairs of values (x and y) represent two quantities that are proportional?

X: 4, 6, 10, 14
Y: 2, 3, 5, 7

Answers

Answer:

No

Step-by-step explanation:

In a proportional relationship, it will be a straight line (it is here) that goes through the origin (it does not).  It would not go through (0.0), but would go through (2, 0)

At the Olympic games, many events have several rounds of competition. One of these events is the men's 100 100100-meter backstroke. The upper dot plot shows the times (in seconds) of the top 8 88 finishers in the final round of the 2012 20122012 Olympics. The lower dot plot shows the times of the same 8 88 swimmers, but in the semifinal round. Which pieces of information can be gathered from these dot plots? (Remember that lower swim times are faster.) Choose all answers that apply: Choose all answers that apply:

Answers

Answer:

The center of the semifinal round distribution is greater than the center of final round distribution.

The variability in the semifinal round distribution is less than variability in the final round distribution.

Step-by-step explanation:

The mean value of each distribution set is not calculates as the center of semifinal round distribution is greater than the final round distribution. MAD Mean Absolute Deviation is calculated from the dotted graph plot, the distribution of semifinal round is less spread out than the final round distribution.

Answer:

correct answer is None of the above i understood nothing the other person was trying to say...

Step-by-step explanation:

mark me brainliest please...

Look for a pattern in the first three diamonds below. For the fourth diamond, explain how you could find the missing numbers if you know the two numbers

Answers

Answer:

See below.

Step-by-step explanation:

The top number is the product (multiplication) of the two side numbers. The bottom number is the sum (addition) of the two side numbers.

I'll do problem a as an example:

Can someone please help me on these 4 questions PLEASE HELP ME!!

Answers

Answer:4 ans x is -32/10

7 ans b  is 6

10 ans x is 25/7

13 ans x is 9/2

Step-by-step explanation:

8= 10x+4010x= 8-4010x= -32x= -32÷10x= -3.2

2b+3b-10= 205b= 20+105b= 30b= 30÷5b= 6

4(3x-1)= 24+5x12x-4= 24+5x12x-5x= 24+46x= 28x= 28÷6x= 4.66

2x+17-9+x= 5x-12x+x-5x= -1-17+93x-5x= -18+92x= -9x= -9÷2x= -4.5

please mark this answer as brainlist


Hey, please help solve the question.​

Answers

Answer:

75%=x-125

90%=x+250

subtract the second from the first

15%=375

100%=?

100%×375/15

100%=2500marked price is 2500

2500+250=2750

90%=2750

100%=?

cost price=3055.56

l= 6 ft, w = 2 ft, h= 5 ft what is the surface area of the rectangular prism​

Answers

it’s literally 6x2x5 (60 sq ft)

104 ft^2

Explanation:

The surface areaa of a rectangular prism is given by the formula:

2 (lw + hl + hw)

l = length

w = width

h = height

Substitute the formula with the givsn dimensions:

2 ([6 x 2] + [5 x 6] + [5 x 2])

= 2 (12 + 30 + 10)

2 (52)

= 104

So the SA is 104 square feet or ft^2

Hope this helps!

What is the x-value of the solution to the system of equations? 5x + 4y = 8 2x − 3y = 17 −3 −2 4 5

Answers

Answer:

            x = 4

Step-by-step explanation:

5x + 4y = 8   ⇒   4y = 8 - 5x  ⇒  y = (8 - 5x)÷4

2x − 3y = 17

2x - 3(8 - 5x)÷4 = 17    

          ×4            ×4    

8x - 3(8 - 5x) = 68

8x - 24 + 15x = 68

          +24       +24

           23x  =  92  

           ÷23      ÷23

               x = 4

Answer:

it is 4

Step-by-step explanation:

i did it on edge hope it helps.:)

how do you find class width pliz help mi​

Answers

Answer:

Class width refers to the difference between the upper and lower boundaries of any class (category).

...

To find the width:Calculate the range of the entire data set by subtracting the lowest point from the highest,Divide it by the number of classes.

Round this number up (usually, to the nearest whole number).

Hope this helps you ❤️

MaRk mE aS braiNliest ❤️

Defined the worst calculate the entire end of day is it by subtracting the lowest points from the highest

-56 divided by -8 find the quotient

Answers

Answer:

7

Step-by-step explanation:

We can see that we have a negative number divided by a negative number, so the negatives cancel each other out.

[tex]56\div8[/tex]

We know that 8 will go into 56 7 times, so [tex]-56\div-8=7[/tex].

Hope this helped!

Answer:

7

Step-by-step explanation:

Match each number to the letter that represents its
position on the number line.

HELP ASAP

Answers

Answer:

Step-by-step explanation:

Answer

answer bellow

Step-by-step explanation:

24, C

25, D

26, B

27, A

What is the answer?

Answers

The answer would be c since the ten is being subtracting
Other Questions
Jean Paul is an interior designer who is working with a difficult client. Part of his design requires that he put 11 colored vases in a row on a shelf. He has 3 identical blue vases, 2 identical green vases, 4 identical red vases, a purple vase and a yellow vase. He has put up 4 different arrangements of the vases that his client complained about. As he begins to put up the fifth arrangement, he wonders how many different arrangements he might have to go through before his client complains about all of them. How many different arrangements could Jean Paul make what is the range of the function on the graph (1,2), Suppose 232subjects are treated with a drug that is used to treat pain and 50of them developed nausea. Use a 0.01significance level to test the claim that more than 20%of users develop nausea. Identify the null and alternative hypotheses for this test. A. Upper H0?: p equals 0.20 Upper H1?: p not equals 0.20B. Upper H0?: p equals 0.20 Upper H1?: p greater than 0.20C. Upper H0?: p greater than 0.20 Upper H1?: p equals 0.20D. Upper H0?: p equals 0.20 Upper H1?: p less than 0.20Identify the test statistic for this hypothesis test. Identify the P-value for this hypothesis test. Identify the conclusion for this hypothesis test.A. Reject Upper H 0. There is sufficient evidence to warrant support of the claim that more than 20?% of users develop nausea.B. Fail to reject Upper H 0. There is sufficient evidence to warrant support of the claim that more than 20?% of users develop nausea.C. Reject Upper H 0. There is not sufficient evidence to warrant support of the claim that more than 20?% of users develop nausea.D. Fail to reject Upper H 0. There is not sufficient evidence to warrant support of the claim that more than 20?% of users develop nausea. Given the function g(x) = x^2 + 5x + 14, determine the average rate of changeof the function over the interval 1 < x < 7. what is one property of viruses that makes it difficult to cure viral infection simplify 3/7into a whole number Answer please!!!!!!!!!!! Point E is on line segment DF. Given DEEF.6 and DF =9, determine the length EF f(x)=6x+2 and g(x)=-9x-5 Find the product of f and g. Cho X l i lng ngu nhin c hm mt dngfx=k(x-1)(4-x) nu x0;4.Tm k, F, EX, P0 Find the slope of the line passing through the points -4,-3 and 8,-9 please help I am not sure about my answer If AC = 40, find the length of JK. Niles believes that the reports are true and does not intend to deceive Prime Business, but does not check the reports before certifying them. Can Niles be held liable to Prime Business Easy Question. Will Mark Brainliest. Many cruise ships use advanced wastewater treatment technologies to treat sewage before they discharge it into the sea. Is the use of non-GMO bacteria to clean this water, as opposed to using chemical additives like chlorine, an example of biotechnology? Why or why not? This is an example of biotechnology because it seeks to solve a societal problem using biological entities. This is an example of biotechnology because metabolic activity of any organism is considered biotechnology. This is not an example of biotechnology because it isn't new, people have been using bacteria to treat wastewater for years. This is not an example of biotechnology because the bacteria are not genetically modified. Find the density of a cube on Earth that weighs 1.5 kg and has a side-length of 10 cm. 0,2,4,0,2,3,2,8,6What is the mean?What is the median?!What is the first quartile (Q1)?!What is the third quartile (Q3)?What is the minimum?What is the maximum?What is the interquartile range of the data?! explain the importance of the Independence of the judiciary Your camera owners manual has information on the functions and locations of the parts of your camera. True or False Seals such as the one pictured above could best be used as early evidence for what historical continuity?A)The existence of trade between regionsB)The spread of religious belief systemsC)The use of art to reinforce political powerD)The codification of legal systems for public display